You are on page 1of 23

PHÉP NGHỊCH ĐẢO

TRONG CHỨNG MINH VUÔNG GÓC

Trần Minh Ngọc


(Học viên Cao học K27 Đại học Sư phạm TP.HCM)

Tống Hữu Nhân


(Sinh viên Đại học Y Dược TP.HCM)

Tóm Tắt

Bài viết tiếp cận một lớp bài toán chứng minh tâm đường tròn nằm trên đường
thẳng vuông góc với đường thẳng cho trước. Dưới góc nhìn của phép nghịch đảo,
từ mô hình tổng quát, ta sẽ xây dựng các phương án khác nhau để từ đó sáng tạo
nên các bài toán thú vị.

1. Mô hình tổng quát

Ta bắt đầu bằng mô hình sau : Xét hai phép nghịch đảo

IIk : A ↔ B, IJh : A ↔ C,

với điểm O là tâm (ABC), và một điểm S bất kỳ. Ta sẽ đi tìm điều kiện để tâm O nằm
trên đường thẳng qua điểm S và vuông góc với đoạn IJ.

Theo định lý bốn điểm kết hợp với phương tích, OS ⊥ IJ tương đương với

IO2 − JO2 = IS 2 − JS 2 ,
PI/(O) − PJ/(O) = IS 2 − JS 2 ,
IA · IB − JA · JC = IS 2 − JS 2 . (∗)

Ta sẽ tìm các phương án để rút gọn đẳng thức (∗) về một cấu hình nào đó, từ đó xây
dựng các bài toán.

2. Các phương án

1
Phép nghịch đảo trong chứng minh vuông góc Trần Minh Ngọc - Tống Hữu Nhân

2.1. Phương án 1

Phương án 1. Ta xây dựng ba điểm D, E, F thuộc đường tròn (S) sao cho

IIk : D ↔ E, IJh : D↔F

Khi đó đường thẳng IJ là trục đẳng phương của hai đường tròn (O) và (S).

Lời giải. Sử dụng tính chất phép nghịch đảo và phương tích, ta có

PI/(O) = IA · IB = ID · IE = PI/(S) ,

nên I thuộc trục đẳng phương của (O) và (S). Tương tự cho J, ta thu được điều phải
chứng minh.

Nhận xét. Khi√k, h > 0,


√ xét trường hợp đặc biệt điểm S là giao điểm của hai đường tròn
nghịch đảo (I, k), (J, h) thì ta xây dựng được D ≡ E ≡ F ≡ S, khi đó
2 2
IIIS : A ↔ B, IJJS : A ↔ C,

và đường thẳng IJ là trục đẳng phương của đường tròn (O) và đường tròn điểm S.

Để hiểu rõ hơn, mời bạn đọc cùng xem một số bài toán sau đây.

Bài toán 1. (Trần Minh Ngọc) Cho tam giác nhọn 4ABC nội tiếp đường tròn (O), có
trực tâm H và trung điểm các cạnh AB, AC lần lượt là M, N . Điểm T bất kỳ trên đường
tròn (O), các tia M T, N T lần lượt cắt đường tròn (AHB), (AHC) tại E, F . Chứng minh
rằng tâm đường tròn (T EF ) nằm trên đường cao AH.

X Y

M N
H
E
O

B F C

2
Trần Minh Ngọc - Tống Hữu Nhân Phép nghịch đảo trong chứng minh vuông góc

Lời giải. Do (O) lần lượt đối xứng với (AHB), (AHC) qua AB, AC nên nếu lấy X, Y đối
xứng với T qua M, N thì X ∈ (AHB), Y ∈ (AHC). Từ đó, nếu gọi J là tâm của (T EF ),
kết hợp phương tích, ta có

M A2 = −M A · M B = −M E · M X = M E · M T = PM/(J) .

Tương tự cho N , ta được M N là trục đẳng phương của (J) và đường tròn điểm A, suy ra
JA ⊥ M N . Mà M N là đường trung bình của 4ABC nên AJ ⊥ BC, hay J ∈ AH.
Nhận xét. Bài toán được xây dựng theo phương án 1 dựa trên hai phép nghịch đảo
MA 2 2
IM : (HAB) ↔ (O), T ↔ E, INN A : (HAC) ↔ (O), T ↔ F,

và đỉnh A là giao điểm của hai đường tròn nghịch đảo (M, M A), (N, N A).

Bài toán 2. (Tống Hữu Nhân) Cho tam giác 4ABC. Đường thẳng qua trực tâm H và
song song cạnh BC cắt các cạnh AB, AC tại E, F . Đường tròn (AE), (AF ) lần lượt cắt
BH, CH lần thứ hai tại M, N . Chứng minh rằng tâm đường tròn (HM N ) nằm trên
đường trung tuyến đỉnh A của tam giác 4ABC.

P Q
K
X M

E J F
H

B C
D

Lời giải. Gọi BK, CL là đường cao và AD là trung tuyến của 4ABC.
Cách 1. Đường thẳng qua A, vuông góc AC, AB lần lượt cắt CH, BH tại X, Y . Do D là
tâm (BLKC), kết hợp hệ thức lượng tam giác vuông và phương tích, ta có

AX 2 − AY 2 = XC · XL − Y B · Y K
= PX/(D) − PY /(D) = DX 2 − DY 2 ,

nên theo định lý bốn điểm, ta được AD ⊥ XY. (1)


Mặt khác, dễ thấy AX, AY lần lượt là tiếp tuyến của (AF ), (AE). Do đó, nếu gọi J là
tâm (HM N ), sử dụng phương tích, ta có

XA2 = XH · XN = PX/(J) .

3
Phép nghịch đảo trong chứng minh vuông góc Trần Minh Ngọc - Tống Hữu Nhân

Tương tự cho Y , ta được XY là trục đẳng phương của (J) và đường tròn điểm A, nên
AJ ⊥ XY. (2)
Từ (1) và (2) ta thu được điều phải chứng minh.
Cách 2. Gọi P, Q, J lần lượt là trung điểm AE, AF, EF . Do JS là đường trung bình của
4AEF nên JP k AF , hay JP ⊥ HM . Mà P là tâm (AEHM ) nên JP là trung trực
HM . Chứng minh tương tự, ta cũng có JQ là trung trực HN , suy ra J là tâm (HM N ).
Từ đó, do EF k BC nên AJ là trung tuyến của 4ABC.
Nhận xét. Bài toán được xây dựng theo phương án 1 dựa trên hai phép nghịch đảo
XA 2 2
IX : C ↔ L, H ↔ N, IYY A : B ↔ K, H ↔ M,

và đỉnh A là giao điểm của hai đường tròn nghịch đảo (X, XA), (Y, Y A).

Bài toán 3. (Trần Minh Ngọc) Cho tam giác 4ABC nội tiếp đường tròn (O) và có
_ _
trực tâm H. Gọi X, Y lần lượt là trung điểm cung AHB, AHC. Chứng minh rằng tâm
đường tròn (OXY ) nằm trên đường phân giác trong góc A của 4ABC.

A
F

E H
M N
X
J
Y O

B C

E0

F0

Lời giải. Gọi E, F lần lượt là trung điểm cung nhỏ AB, AC của (O) và kẻ đường kính
EE 0 , F F 0 của (O). Bằng biến đổi góc, ta dễ dàng chứng minh EF vuông góc với phân
giác trong đỉnh A của 4ABC. (1)
Gọi M, N là trung điểm AB, AC thì dễ thấy M, N cũng lần lượt là trung điểm EX, F Y .
Từ đó, nếu gọi J là tâm (OXY ), kết hợp hệ thức lượng tam giác vuông và phương tích,
ta có
EA2 = EM · EE 0 = EX · EO = PE/(J) .
Tương tự cho F , ta được EF là trục đẳng phương của (J) và đường tròn điểm A, nên
AJ ⊥ EF. (2)
Từ (1) và (2) ta thu được điều phải chứng minh.

4
Trần Minh Ngọc - Tống Hữu Nhân Phép nghịch đảo trong chứng minh vuông góc

Nhận xét. Bài toán được xây dựng theo phương án 1 dựa trên hai phép nghịch đảo
2 2
IEEA : M ↔ E 0 , O ↔ X, INN A : N ↔ F 0 , O ↔ Y,

và đỉnh A là giao điểm của hai đường tròn nghịch đảo (E, EA), (F, F A).

Bài toán 4. (Tống Hữu Nhân) Cho tam giác 4ABC nội tiếp đường tròn (O). Gọi
_ _ _
D, E, F lần lượt là trung điểm các cung CAB, ABC, BCA. Đường thẳng DF cắt cạnh
AB tại M , đường thẳng DE cắt cạnh AC tại N . Chứng minh rằng tâm đường tròn
(DM N ) nằm trên đường phân giác trong đỉnh A của 4ABC.

D
J

O
F

B C

Lời giải. Dựa theo tính chất phân giác và trung điểm cung, dễ dàng chứng minh AD k EF
(cùng vuông góc với phân giác trong đỉnh A) và CF k DE (cùng vuông góc với phân giác
trong đỉnh C). Do đó

1 _ 1 _ 1 _
∠ADE = sd AE = sd DF = sd CE = ∠CAE,
2 2 2
hay EA là tiếp tuyến của (AN D), tương tự thì F A là tiếp tuyến của (AM D). Từ đó, nếu
gọi J là tâm (DM N ), sử dụng phương tích, ta có

EA2 = EN · ED = PE/(J) .

Tương tự cho F , ta được EF là trục đẳng phương của (J) và đường tròn điểm A, nên
AJ ⊥ EF . Từ đây, ta dễ dàng thu được điều phải chứng minh.

Nhận xét. Bài toán được xây dựng theo phương án 1 dựa trên hai phép nghịch đảo
2 2
IEEA : D ↔ N, IFF A : D ↔ M,

và đỉnh A là giao điểm của hai đường tròn nghịch đảo (E, EA), (F, F A).

5
Phép nghịch đảo trong chứng minh vuông góc Trần Minh Ngọc - Tống Hữu Nhân

Bài toán 5. (Trần Minh Ngọc) Cho tam giác 4ABC có các đường phân giác trong
AD, BE, CF và P là giao điểm hai tiếp tuyến tại B, C của đường tròn (ABC). Gọi
M, N là trung điểm các đoạn CE, BF . Đường thẳng P M, P N lần lượt cắt các đường
tròn (P BE), (P CF ) tại Q, R. Chứng minh rằng tâm đường tròn (P QR) nằm trên phân
giác AD.

Y E
F
Q
R

N M

B C
D
J

_ _
Lời giải. Giả sử BE, CF cắt lại (ABC) tại X, Y thì X, Y lần lượt là trung điểm AC, AB
và AD ⊥ XY . Bằng biến đổi góc, dễ thấy 4XCE ∼ 4XBC, mà XM là đường trung
tuyến của 4XCE nên là đường đối trung của 4XBC, hay P ∈ XM . Tương tự, ta suy
ra P = XM ∩ Y N.
_
Gọi J là tâm (P QR). Do X là trung điểm AC, kết hợp phương tích, ta có
XA2 = XB · XE = XP · XQ = PX/(J) .
Tương tự cho Y , ta được XY là trục đẳng phương của (J) và đường tròn điểm A, nên
AJ ⊥ XY , mà AD ⊥ XY , nên J ∈ AD.
Nhận xét. Bài toán được xây dựng theo phương án 1 dựa trên hai phép nghịch đảo
XA 2 2
IX : P ↔ Q, B ↔ E INN A : P ↔ R, C ↔ F,
và đỉnh A là giao điểm của hai đường tròn nghịch đảo (X, XA), (Y, Y A).

Bài toán 6. (Trần Minh Ngọc) Cho tam giác 4ABC nội tiếp đường tròn (O), ngoại
tiếp đường tròn (I) và phân giác AI cắt lại đường tròn (O) tại điểm S. Một đường
thẳng qua tâm nội I lần lượt cắt các cạnh BC, CA, AB tại D, E, F . Đường thẳng SD
cắt đường tròn (O) lần nữa tại điểm T . Chứng minh rằng tâm đường tròn (T EF ) nằm
trên phân giác AI.

6
Trần Minh Ngọc - Tống Hữu Nhân Phép nghịch đảo trong chứng minh vuông góc

A
X

Y
E

F I

B C
D

_ _
Lời giải. BI, CI lần lượt cắt lại (O) tại X, Y thì X, Y lần lượt là trung điểm AC, AB.
Bằng biến đổi góc, dễ chứng minh AI ⊥ XY.

Giả sử E 0 = T X ∩ AC, áp dụng định lý Pascal cho 6 điểm A, C, B, X, T, S thì ta được


D, I, E 0 thẳng hàng hay E 0 ≡ E, tương tự suy ra T = XE ∩ Y F . Từ đó, do X là trung
_
điểm AC, nên nếu gọi J là tâm (T EF ), ta có

XA2 = XE · XT = PX/(J) .

Tương tự cho Y , ta được XY là trục đẳng phương của (J) và đường tròn điểm A, nên
AJ ⊥ XY , mà AI ⊥ XY nên J ∈ AI.

Bài toán 7. (Trần Minh Ngọc) Cho tam giác 4ABC có phân giác BE, CF cắt nhau
tại tâm nội I. Đường tròn (I, IA) cắt cạnh BC tại M, N . Chứng minh rằng tâm đường
tròn (IM N ) nằm trên đường cao đỉnh A của 4AEF.

Lời giải. Không mất tính tổng quát, giả sử các điểm trên BC theo thứ tự B, M, N, C thì
do BI là phân giác ∠ABC nên

∠IM N = ∠IN M = ∠BAI,

hay 4BAN cân tại B và ABM I nội tiếp.


_
Gọi X = AM ∩ BE, Y = AN ∩ CF . Ta có I là trung điểm AM của (ABM ) nên
IA2 = IB · IX, kết hợp phương tích, tương đương với

BI · BX = IB 2 − IA2 = PB/(I,IA) = BM · BN ,

hay IXM N nội tiếp. Tương tự, suy ra I, X, Y, M, N cùng thuộc một đường tròn.

7
Phép nghịch đảo trong chứng minh vuông góc Trần Minh Ngọc - Tống Hữu Nhân

E
F

Y
I
X

B M C
J N

Từ đó, sử dụng góc nội tiếp, ta được

∠EAI = ∠IN M = ∠AXI,

hay EA là tiếp tuyến của (AIX). Từ đó, nếu gọi J là tâm (IXY ), sử dụng phương tích,
ta có
EA2 = EI · EX = PE/(J) .
Tương tự cho F , ta được EF là trục đẳng phương của (J) và đường tròn điểm A, nên
AJ ⊥ EF , hay J nằm trên đường cao đỉnh A của 4AEF.

Nhận xét. Bài toán được xây dựng theo phương án 1 dựa trên hai phép nghịch đảo
2 2
IEEA : I↔X IFF A : I ↔ Y,

và đỉnh A là giao điểm của hai đường tròn nghịch đảo (E, EA), (F, F A).

2.2. Phương án 2

Phương án 2. Ta xây dựng hai điểm M, N thỏa

IIk : S ↔ M, IJh : S↔N

Khi đó đẳng thức (∗) xảy ra khi và chỉ khi bốn điểm I, J, M, N cùng thuộc một đường
tròn.

8
Trần Minh Ngọc - Tống Hữu Nhân Phép nghịch đảo trong chứng minh vuông góc

O
C
B

I J
S

N
M

Lời giải. Sử dụng tính chất phép nghịch đảo, ta có

IS 2 − JS 2 = IA · IB − JA · JC = IS · IM − JS · JN ,

Do đó, (∗) tương đương với SI · SM = SJ · SN , hay I, J, M, N cùng thuộc một đường
tròn.

Nhận xét. Nếu nhìn theo góc độ của phương án 1, xét hai phép nghịch đảo

IIk : A ↔ B, S ↔ M IJh : A ↔ C, S ↔ N,

thì ta được tâm đường tròn (SM N ) cũng nằm trên đường thẳng OS và đường thẳng IJ
là trục đẳng phương của hai đường tròn (O) và (SM N ).

Để hiểu rõ hơn, mời bạn đọc cùng xem các bài toán sau đây.

Bài toán 8. Cho tam giác 4ABC và có trực tâm H. Các đường tròn (HAB), (HAC)
lần lượt cắt các cạnh AC, AB lần thứ hai tại các điểm M, N . Chứng minh rằng tâm
đường tròn (HM N ) nằm trên đường thẳng Euler của tam giác 4ABC.

(APMO 2010)

Lời giải. Gọi O, O0 lần lượt là tâm (ABC) và (HM N ) và X = HM ∩ BC, Y = HN ∩ AC.
Sử dụng phương tích, ta có

PX/(O0 ) = XH · XM = XA · XC = PX/(O) ,

PY /(O0 ) = XH · Y N = Y A · Y B = PY /(O) ,
nên XY là trục đẳng phương của (O0 ) và (O), suy ra XY ⊥ OO0 . (1)

9
Phép nghịch đảo trong chứng minh vuông góc Trần Minh Ngọc - Tống Hữu Nhân

O0
O

B C

Mặt khác, sử dụng góc nội tiếp, ta có

∠XM A = ∠ABH = ∠ACH = ∠Y N A,

suy ra XM N Y nội tiếp. Từ đó, sử dụng phương tích, ta được

HX 2 − HY 2 = HX · (HM + M X) − HY · (HN + N Y )
= HX · HM + XH · XM − HY · HN − Y H · Y N
= PH/(XM N Y ) + XA · XC − PH/(XM N Y ) − Y A · Y B
= PX/(O) − PY /(O) = OX 2 − OY 2 ,

nên theo định lý bốn điểm, ta được XY ⊥ OH. (2)

Từ (1) và (2) suy ra O, H, O0 thẳng hàng, ta thu được điều phải chứng minh.

Nhận xét. Ý (1) của bài toán được xây dựng theo phương án 1 dựa trên hai phép nghịch
đảo
k
IX : A ↔ B, H ↔ M, IYh : A ↔ C, H ↔ N,

Ý (2) của bài toán được xây dựng theo phương án 2 dựa trên hai phép nghịch đảo
k
IX : A ↔ B, H ↔ M, IYh : A ↔ C, H ↔ N,

và tứ giác XM N Y nội tiếp.

10
Trần Minh Ngọc - Tống Hữu Nhân Phép nghịch đảo trong chứng minh vuông góc

Bài toán 9. Dựng về phía ngoài tam giác 4ABC hai tam giác 4ABX, 4ACY cân
tại đỉnh A và đồng dạng với nhau. Gọi Z là giao điểm của hai đường thẳng BY, CX.
Chứng minh rằng tâm đường tròn (BZC) nằm trên đường cao đỉnh A của 4AXY.

N A

X
Z

B C

Lời giải. Do 4ABX ∼ 4ACY và cân tại A nên AX = AB, AC = AY và

∠XAC = ∠BAC + ∠XAB = ∠BAC + ∠Y AC = ∠Y AB,

suy ra 4AXC = 4ABY . Từ đó ∠AXC = ∠ABY , hay AXBZ nội tiếp. Tương tự thì
AY CZ cũng nội tiếp.
Gọi M, N lần lượt là giao điểm thứ hai của AX, AY với (ACZ), (ABZ). Sử dụng góc nội
tiếp, kết hợp 4ABX ∼ 4ACY , ta có

∠XM Y = 180◦ − ∠ACY = 180◦ − ∠ABX = ∠XN Y ,

hay XY M N nội tiếp. Từ đó, nếu gọi J là tâm (BZC), sử dụng phương tích, ta được

AX 2 − AY 2 = AX · (AM + M X) − AY · (AN + N Y )
= AX · AM + XA · XM − AY · AN − Y A · Y N
= PA/(XY M N ) + XZ · XC − PA/(XY M N ) − Y Z · Y B
= PX/(J) − PY /(J) = JX 2 − JY 2 ,

nên theo định lý bốn điểm, ta được AJ ⊥ XY , hay J nằm trên đường cao đỉnh A của
4AXY.

11
Phép nghịch đảo trong chứng minh vuông góc Trần Minh Ngọc - Tống Hữu Nhân

Nhận xét. Bài toán được xây dựng theo phương án 2 dựa trên hai phép nghịch đảo
k
IX : Z ↔ C, A ↔ M, IYh : Z ↔ B, A ↔ N,
và tứ giác XY M N nội tiếp.

Bài toán 10. (Trần Minh Ngọc) Cho tam giác 4ABC và hai điểm E, F thuộc cung
_ _
nhỏ AB, AC. Lấy M, N lần lượt đối xứng với các điểm E, F qua các cạnh AB, AC. Gọi
P là giao điểm cả hai đường thẳng BM, CN . Chứng minh rằng tâm đường tròn (M N P )
nằm trên đường cao đỉnh A của 4ABC.

E
P
X M N

B C

Lời giải. Gọi X, Y lần lượt là giao điểm thứ hai của AB, AC với (AP M ), (AP N ). Sử
dụng tính đối xứng và góc nội tiếp, ta chứng minh được X, Y, P thẳng hàng và BXY C
nội tiếp. Từ đó, nếu gọi J là tâm (BZC), sử dụng phương tích, ta được
AB 2 − AC 2 = AB · (AX + XB) − AC · (AY + Y C)
= AB · AX + BA · BX − AC · AY − CA · CY
= PA/(BXY C) + BP · BM − PA/(BXY C) − CP · CN
= PB/(J) − PC/(J) = JB 2 − JC 2 ,

nên theo định lý bốn điểm, ta được AJ ⊥ BC, hay J nằm trên đường cao đỉnh A của
4ABC. Kết luận của bài toán vẫn đúng nếu thay M, N lần lượt đối xứng với các điểm
E, F qua trung điểm các cạnh AB, AC.
Nhận xét. Bài toán được xây dựng theo phương án 2 dựa trên hai phép nghịch đảo
IBk : P ↔ M, A ↔ X, ICh : P ↔ N, A ↔ Y,
và tứ giác BXY C nội tiếp.

12
Trần Minh Ngọc - Tống Hữu Nhân Phép nghịch đảo trong chứng minh vuông góc

Bài toán 11. (Trần Minh Ngọc) Cho tam giác 4ABC và hai điểm M, N nằm trên
các cạnh AB, AC sao cho BM = CN . Gọi S là giao điểm khác A của hai đường tròn
(ABN ), (ACM ). Chứng minh rằng tâm các đường tròn (ABC), (AM N ) lần lượt nằm
trên đường cao đỉnh S của 4SM N, 4SBC.

N
O

B C

S
Y X

Lời giải. Gọi O, J lần lượt là tâm (ABC), (AM N ). Giả sử SB, SC lần lượt cắt lại
(ASC), (ASB) tại X, Y . Ta có

∠SM B = ∠SCN , ∠SN C = ∠SBM , BM = CN,

nên 4SBM = 4SN C, suy ra d(S, AB) = d(S, AC), hay AS là phân giác ∠BAC. Sử
dụng góc nội tiếp, ta được

∠N AY = ∠N SC = ∠M SB = ∠M AX,

hay AX, AY đẳng giác góc A. Từ đó

∠SBY = ∠SAY = ∠SAX = ∠SCX,

hay BCXY nội tiếp. Sử dụng phương tích, ta được

SB 2 − SC 2 = SB · (SX + XB) − SC · (SY + Y C)


= SB · SX + BS · BX − SC · SY − CS · CY
= PS/(BCXY ) + BA · BM − PS/(BCXY ) − CA · CN
= PB/(J) − PC/(J) = JB 2 − JC 2 ,

nên theo định lý bốn điểm, ta được SJ ⊥ BC.

13
Phép nghịch đảo trong chứng minh vuông góc Trần Minh Ngọc - Tống Hữu Nhân

Mặt khác, do 4SBM = 4SN C nên


SM 2 − SN 2 = SC 2 − SB 2 = CA · CN − BA · BM
= (CN + N A) · CN − (BM + M A) · BM
= CN 2 − BM 2 + M A · M B − N A · N C
= PM/(O) − PN/(O) = OM 2 − ON 2 ,
nên theo định lý bốn điểm, ta được SO ⊥ M N.
Nhận xét. Bài toán được xây dựng theo phương án 2 dựa trên hai phép nghịch đảo
IBk : A ↔ M, S ↔ X, ICh : A ↔ N, S ↔ Y,
và tứ giác BCXY nội tiếp.

2.3. Phương án 3

Phương án 3. Ta xây dựng hai điểm M, N thỏa

IIk : J ↔ M, IJh : I ↔ N.

Khi đó đẳng thức (∗) xảy ra khi và chỉ khi tam giác 4SM N cân tại đỉnh S.

O
C

I M D N J

Lời giải. Sử dụng tính chất phép nghịch đảo, ta có


IS 2 − JS 2 = IA · IB − JA · JC = IJ · IM − JI · JN = IJ · (IM + JN ).
Mặt khác, gọi D là hình chiếu của S lên IJ, ta được
IS 2 − JS 2 = ID2 − JD2 = IJ · (ID + JD).
Suy ra (∗) tương đương với DM + DN = 0, hay D là trung điểm M N , hay 4SM N cân
tại S.

14
Trần Minh Ngọc - Tống Hữu Nhân Phép nghịch đảo trong chứng minh vuông góc

Bài toán 12. (Trần Minh Ngọc) Cho tam giác 4ABC có đường cao AD, BE, CF
và trung tuyến AM . Lấy B 0 , C 0 đối xứng với các đỉnh B, C qua điểm D. Từ điểm T
bất kỳ thuộc trung tuyến AM , kẻ đường thẳng T B, T C lần lượt cắt các đường tròn
(BF B 0 ), (CEC 0 ) lần thứ hại tại X, Y . Chứng minh rằng tâm đường tròn (T XY ) nằm
trên đường cao AD.

Y E
J

T
F
X

C0 B D B0 M C

Lời giải. Do D, M lần lượt là trung điểm BB 0 , BC nên sử dụng phương tích, ta có

BA · BF = BC · BD = BM · BB 0 ,

nên AM B 0 F nội tiếp. Từ đó, sử dụng góc nội tiếp, ta được

∠M AF = ∠BB 0 F = ∠BXF ,

hay AT XF nội tiếp, tương tự thì AT Y E cũng nội tiếp. Từ đó, nếu gọi J là tâm (T XY ),
sử dụng phương tích, ta có

JB 2 − JC 2 = PB/(J) − PC/(J) = BT · BX − CT · CY
= BA · BF − CA · CE
= BC · BD − CB · CD = BD2 − CD2 ,

nên theo định lý bốn điểm, ta được JD ⊥ BC, hay J ∈ AD.

Nhận xét. Bài toán được xây dựng theo phương án 3 dựa trên hai phép nghịch đảo

IBk : T ↔ X, C ↔ D, ICh : T ↔ Y, B ↔ D.

Bài toán 13. Cho tam giác 4ABC và một điểm D bất kỳ nằm trên trung tuyến AM .
Đường tròn (ADB), (ADC) lần lượt cắt cạnh AC, AB lần thứ hai tại E, F . Chứng minh
rằng tâm đường tròn (AEF ) nằm trên trung trực đoạn BC.

15
Phép nghịch đảo trong chứng minh vuông góc Trần Minh Ngọc - Tống Hữu Nhân

J
A

B X M Y C

Lời giải. Giả sử BC lần lượt cắt (ADB), (ADC) lần thứ hai tại X, Y . Do M là trung
điểm BC và thuộc trục đẳng phương của (ADB), (ADC) nên dễ dàng chứng minh M
cũng là trung điểm XY . Từ đó, nếu gọi J là tâm của AEF , sử dụng phương tích, ta có

PB/(J) = BA · BF = BY · BC = CX · CB = CA · CE = PB/(J) .

Từ đây suy ra JB = JC, hay J thuộc trung trực BC.

Nhận xét. Bài toán được xây dựng theo phương án 3 dựa trên hai phép nghịch đảo

IBk : A ↔ F, C ↔ Y, ICh : A ↔ E, B ↔ X,

và tam giác ‘suy biến’ 4M XY cân tại đỉnh M.

Bài toán 14. (Tống Hữu Nhân) Cho tam giác 4ABC có đường cao BE, CF . Đường
tròn qua A, B và tiếp xúc cạnh BC cắt cạnh AC tại X; đường tròn qua A, C và tiếp
xúc cạnh BC cắt cạnh AB tại Y . Gọi M, N lần lượt là trung điểm các đoạn BY, CX.
Chứng minh rằng

(a) Tâm các đường tròn (AXY ), (AM N ) nằm trên trung trực cạnh BC.

(b) Tâm các đường tròn (AEM ), (AF N ) và đường tròn Euler của 4ABC thẳng hàng.

Lời giải. (a) Gọi I, J lần lượt là tâm (AXY ), (AM N ) và S là giao điểm khác A của
(ABX), (ACY ). Do BC là tiếp tuyến chung của (ABX), (ACY ) nên nếu gọi T là trung
điểm BC, sử dụng phương tích, ta có

PT /(ABX) = T B 2 = T C 2 = PT /(ACY ) ,

hay T thuộc trục đẳng phương AS của (ABX) và (ACY ). Áp dụng bài toán 13 cho S
nằm trên trung tuyến AT , ta được I nằm trên trung trực BC.

16
Trần Minh Ngọc - Tống Hữu Nhân Phép nghịch đảo trong chứng minh vuông góc

A I

J1
J

X
M J2 E

O
N
S
F E

B D T C

Mặt khác, do M là trung điểm AY nên sử dụng phương tích, ta có

1 1
BA · BM = · BA · BY = · BC 2 = BC · BT ,
2 2

hay ACT M nội tiếp, tương tự thì ABT N cũng nội tiếp. Lại áp dụng bài toán 13 cho T
nằm trên trung tuyến AT , ta được J nằm trên trung trực BC.

(b) Kẻ đường cao AD của 4ABC. Gọi J1 , J2 , E lần lượt là tâm (AEM ), (AF N ) và đường
tròn Euler của 4ABC. Sử dụng phương tích, ta có

J1 B 2 − J1 C 2 = PB/(J1 ) − PC/(J1 ) = BA · BM − CA · CE
= BC · BT − CB · CD
= BD · BT − CT · CD = PB/(E) − PC/(E) = EB 2 − EC 2 ,

nên theo định lý bốn điểm, ta được J1 E ⊥ BC. Tương tự thì J2 E ⊥ BC, suy ra J1 , J2 , E
thẳng hàng.

Nhận xét. Ý (b) của bài toán được xây dựng theo phương án 3 dựa trên hai phép nghịch
đảo
IBk : A ↔ M, C ↔ T, ICh : A ↔ E, B ↔ D,

và tam giác 4EDT cân tại đỉnh E.

17
Phép nghịch đảo trong chứng minh vuông góc Trần Minh Ngọc - Tống Hữu Nhân

2.4. Phương án 4

Phương án 4. Ta xây dựng hai điểm M, N thỏa


2 2
IIIS : A ↔ M, IJJS : A ↔ N.

Khi đó đẳng thức (∗) xảy ra khi và chỉ khi IA · BM = JA · CN .

A P1

N
B Y

P2
M

I J

Lời giải. Sử dụng tính chất phép nghịch đảo, ta có


IA · IB − JA · JC = IS 2 − JS 2 = IA · IM − JA · JN ,
tương đương với IA · BM = JA · CN . (∗∗)

Ta có nhiều cách để xây dựng các điểm M, N thỏa mãn đẳng thức (∗∗), chẳng hạn nếu gọi
P là tâm vị tự quay biến đoạn BM thành đoạn CN (có hai điểm P như vậy, lần lượt là
điểm Miquel của các tứ giác toàn phần (AB, AC, BC, M N ) và (AB, AC, BN, CM )) thì
(∗∗) sẽ tương đương với AP song song hoặc chia đôi đoạn IJ (tùy theo vị trí điểm P ).
Nhận xét. Nếu nhìn theo góc độ của phương án 1, xét hai phép nghịch đảo
2 2
IIIS : A ↔ M, IJJS : A ↔ N,
và đỉnh A là giao điểm của hai đường tròn nghịch đảo (I, IS), (J, JS), ta được tâm đường
tròn (AM N ) cũng nằm trên đường thẳng OS và đường thẳng IJ là trục đẳng phương
của hai đường tròn (AM N ) và đường tròn điểm S.

Bài toán 15. (Trần Minh Ngọc) Cho tam giác 4ABC nội tiếp đường tròn (O) có trực
tâm H. Gọi K, L lần lượt là trực tâm các tam giác 4OAB, 4OAC và M là trung điểm
đoạn AH. Đường thẳng OK, OL lần lượt cắt đường tròn (OM L), (OM K) lần thứ hai
tại X, Y . Chứng minh rằng tâm các đường tròn (OKL), (OXY ) nằm trên đường cao
AH.

18
Trần Minh Ngọc - Tống Hữu Nhân Phép nghịch đảo trong chứng minh vuông góc

J
A
Y
X
M

Q
P
H
K
O

B C

Lời giải. Gọi P, Q lần lượt là trung điểm AB, AC và I, J là tâm (OKL), (OXY ). Sử
dụng tính chất trực tâm, kết hợp P trung điểm AB, ta có

P A2 = −P A · P B = P O · P K = PP/(I) .

Tương tự cho Q, ta được P Q là trục đẳng phương của (I) và đường tròn điểm A, nên
AI ⊥ P Q. Mà AH ⊥ P Q nên I ∈ AH.
Sử dụng góc nội tiếp, ta có

∠M XK = ∠M LY , ∠M KX = ∠M Y L,

nên 4M XK ∼ 4M LY . Kết hợp OP M Q là hình bình hành, ta được


XK d(M, OP ) OQ
= = .
YL d(M, OQ) OP

Từ đây, sử dụng phương tích, ta có


AP 2 − AQ2 = P O · P K − QO · QL
= P O · (P X + XK) − QO · (QY + Y K)
= P O · P X + P O · XK − QO · QY − QO · Y L
= PP/(J) − PQ/(J) = JP 2 − JQ2 ,

nên theo định lý bốn điểm, ta được AJ ⊥ P Q, mà AH ⊥ P Q nên J ∈ AH.


Nhận xét. Bài toán được xây dựng theo phương án 1 và phương án 4 đều dựa trên hai
phép nghịch đảo
2 2
IPP A : O ↔ K, IQQA : O ↔ L,
với đỉnh A là giao điểm của hai đường tròn nghịch đảo (P, P A), (Q, QA) và đường thẳng
OM chia đôi đoạn P Q.

19
Phép nghịch đảo trong chứng minh vuông góc Trần Minh Ngọc - Tống Hữu Nhân

Bài toán 16. (Trần Minh Ngọc) Cho tam giác 4ABC có đường tròn nội tiếp (I) tiếp
xúc cạnh BC tại điểm D. Đường tròn (D, DI) cắt lại đường thẳng IB, IC tại X, Y . Tia
phân giác góc ∠XDY cắt các đường thẳng IB, IC tại các điểm M, N . Chứng minh rằng
tâm đường tròn (IM N ) nằm trên đường thẳng AD.

Lời giải. (I) tiếp xúc AB, AC tại E, F.

Cách 1. Giả sử AD, ID lần nữa tại D0 , T . Gọi P, Q lần lượt là giao điểm của D0 T và
BI, CI. Do DT là đường kính của (I) nên AD ⊥ P Q tại D0 . (1)

Mặt khác, ta có DED0 F điều hòa nên BC, EF và tiếp tuyến của (I) tại D0 đồng quy tại
S, suy ra IS ⊥ AD hay IS k P Q. Mà do AD, BE, CF đồng quy nên

I(P QT S) = I(BCDS) = −1.

Từ đó suy ra T là trung điểm P Q.

T
D0 E
Q

F N J
Y
I

S C
B D

Do DM là phân giác ∠XDY , kết hợp góc ở tâm, ta có

1
∠M DX = · ∠XDY = ∠N IX,
2

hay IDXN nội tiếp, tương tự thì IDY M cũng nội tiếp. Từ đó, sử dụng góc nội tiếp, dễ
dàng chứng minh 4DM X ∼ 4DY N . Kết hợp T là trung điểm P Q, ta được

MX d(D, IP ) IQ
= = .
NY d(D, IQ) IP

20
Trần Minh Ngọc - Tống Hữu Nhân Phép nghịch đảo trong chứng minh vuông góc

Từ đó, nếu gọi J là tâm (IM N ), sử dụng phương tích, ta có

DP 2 − DQ2 = PP/(D,DI) − PQ/(D,DI) = P I · P X − QI · QY


= P I · (P M + M X) − QI · (QN + N Y )
= P I · P M + P I · M X − QI · QN − QI · N Y
= PP/(J) − PQ/(J) = JP 2 − JQ2 ,

nên theo định lý bốn điểm, ta được DJ ⊥ P Q. (2)


Từ (1) và (2), ta thu được điều phải chứng minh.
Cách 2. (Nguyễn Hoàng Danh, 12CT, chuyên Lê Hồng Phong, Tp.HCM) Gọi J là tâm
(IM N ). Yêu cầu bài toán tương đương với chứng minh DJ là đường đối trung của 4DEF.

F N Q
J I

Y
P S
M
T
X

B D C

Thật vậy, do DM là phân giác ∠XDY , kết hợp góc ở tâm, ta có


1
∠M DX = · ∠XDY = ∠N IX,
2
hay IDXN nội tiếp, tương tự thì IDY M cũng nội tiếp. Từ đó, sử dụng góc nội tiếp, dễ
dàng chứng minh 4DM X ∼ 4DY N.
Từ đó, gọi P, Q, S, T lần lượt là trung điểm IM, IN, DE, DF , ta có
d(J, DE) QS NY DS DE
= = = = ,
d(J, DF ) PT MX DT DF

nên DJ là đường đối trung của 4DEF.

21
Phép nghịch đảo trong chứng minh vuông góc Trần Minh Ngọc - Tống Hữu Nhân

2.5. Phương án 5

Phương án 5. Ta xây dựng hai điểm M, N thỏa


2 2
IIIS : B ↔ M, IJJS : C ↔ N.

Khi đó đẳng thức (∗) xảy ra khi và chỉ khi IB · AM = JC · AN .

P1

O C

I J

P2

Lời giải. Sử dụng tính chất phép nghịch đảo, ta có

IA · IB − JA · JC = IS 2 − JS 2 = IB · IM − JC · JN ,

tương đương với IB · AM = JC · AN . (∗ ∗ ∗)

Ta có nhiều cách để xây dựng các điểm M, N thỏa mãn đẳng thức (∗ ∗ ∗), chẳng hạn nếu
gọi P là tâm vị tự quay biến đoạn BI thành đoạn CJ (có hai điểm P như vậy, lần lượt là
điểm Miquel của các tứ giác toàn phần (BC, IJ, BI, CJ) và (AB, AC, BJ, CI)) thì (∗ ∗ ∗)
sẽ tương đương với AP song song hoặc chia đôi đoạn M N (tùy theo vị trí điểm P ).

Bài toán 17. (Trần Minh Ngọc) Cho tam giác 4ABC có đường đối trung cắt đường
tròn ngoại tiếp (O) tại D và tiếp tuyến tại đỉnh A của đường tròn (O) cắt cạnh BC tại
T . Đường thẳng bất kỳ qua T , cắt các cạnh AB, AC tại E, F . Đường tròn qua D, E, tiếp
xúc DB cắt lại cạnh AB tại M ; đường tròn qua D, F , tiếp xúc DC cắt lại cạnh AC tại
N . Chứng minh rằng tâm đường tròn (AM N ) nằm trên đường cao đỉnh D của 4DBC.

22
Trần Minh Ngọc - Tống Hữu Nhân Phép nghịch đảo trong chứng minh vuông góc

F
M

E N

T B C

Lời giải. Do AD là đường đối trung của 4ABC, nên ABDC điều hòa, hay T D là tiếp
tuyến tại D của (O). Từ đó, kết hợp định lý Menelaus, ta có

DB 2 TB F A EB
2 = = · ,
DC TC F C EA
tương đương với
AE CD2 BE CN
= 2 · = .
AF BD CF BM
Từ đó, nếu gọi J là tâm (AM N ), sử dụng phương tích, ta có

DB 2 − DC 2 = BE · BM − CF · CN
= (BA + AE) · BM − (CA + AF ) · CN
= BA · BM + AE · BM − CA · CN − AF · CN
= PB/(J) − PC/(J) = JB 2 − JC 2 ,

nên theo định lý bốn điểm, ta được DJ ⊥ BC.

Trên đây, chúng tôi đã tiếp cận một lớp bài toán chứng minh tâm đường tròn nằm trên
đường thẳng vuông góc với đường thẳng cho trước dưới góc nhìn của phép nghịch đảo.
Qua bài viết nhỏ này, hy vọng đã đem đến cho bạn đọc chút cảm nhận về vẻ đẹp độc đáo
của hình học và nhận được những ý kiến đóng góp quý báu để bài viết hoàn thiện hơn.

Tài liệu tham khảo

[1] AoPS : http://artofproblemsolving.com/

23

You might also like